LSAT and Law School Admissions Forum

Get expert LSAT preparation and law school admissions advice from PowerScore Test Preparation.

 Adam Tyson
PowerScore Staff
  • PowerScore Staff
  • Posts: 5153
  • Joined: Apr 14, 2011
|
#46950
When examining the stimulus above, I realize Premise 1 and the Conclusion is the contrapositive of Premise 2, which is "NOT treated by stress-reduction :arrow: NOT caused by stress." This is very similar to "Some HBP treated by medicine" (premise 1) :arrow: "Some HBP not caused by stress" (conclusion).
It looks to me like in this analysis you are making the same assumption that the author made, blueballoon, and that is that "treated by medicine" means "not treated by stress reduction". In other words, you are assuming, as our author did, that medicine doesn't reduce stress! It's not a solid argument unless you also make that same assumption! Since that is what is missing, that is what we need in the correct answer. E provides that, and D does not.
 nusheenaparvizi
  • Posts: 22
  • Joined: Mar 14, 2020
|
#74645
Hi,

I am having a lot of trouble trying to understand how negating answer choice D does not attack the argument?

The way I am trying to understand and justify answer choice E is that medicine and reducing stress are two separate treatments, therefore if an illness is caused by reducing stress, then it can't be treated through medicine (going back to the stimulus that some cases of high BP are not caused by stress so they must be treated with medicine). So I can kind of make sense of why E is correct, especially when you negate it and say "medicine used to treat high BP DOES itself reduce stress" which would attack the argument in the stimulus.

But even though I can understand why E is correct, I cannot unsee D because that answer choice still stands out to me and I want to understand why D is fundamentally wrong so that in the future I can catch the flaws in an answer choice and not fall under the trap.

Negating D would be "all conditions treated by medicines are treated through reducing stress," and I see that this too like E attacks the argument? When I diagram the conditional negation of D I get:

treatable with medicine :arrow: also treatable through reducing stress.

I am genuinely just running myself in circles frustrated that no matter how long I stare at this question and your explanations I feel like I get 1 step closer to understanding and then I lose it completely :cry:

I am sorry if it feels repetitive but can you please explain in the most simplest terms why D is fundamentally incorrect and E is a better assumption? They both feel like attacks on the argument when I use the assumption negation technique.

Thank you,
Nusheena
 Jeremy Press
PowerScore Staff
  • PowerScore Staff
  • Posts: 1000
  • Joined: Jun 12, 2017
|
#74673
Hi Nusheena,

I think the best, and simplest, thought process to follow to understand why answer choice D's negation (which you've correctly identified as "All conditions that are treated effectively by medicines are also treatable through the reduction of stress") does not attack the argument is to take these two steps: first, consider how that negation relates to, and changes your understanding of, the information in the first sentence. Next, consider whether that changed understanding invalidates the conclusion.

Step One
The negated form of answer choice D, if true, would change our understanding of the first sentence of the stimulus. It would require us to say that those cases of high blood pressure that can be treated effectively with medicine can also be treated through the reduction of stress.

Step Two
What does that changed understanding of the first sentence mean for the conclusion? Does saying something can be treated through the reduction of stress necessarily mean that such a condition is caused by stress? Not necessarily, for the reasons that Adam notes in his very helpful post above. Just because a reduction of one thing (stress) is correlated with reduction of another thing (level of blood pressure) doesn't necessarily mean there is a causal relationship between those two things. A third thing might be causing both, or there might be some other explanation for the correlation that is not immediately clear. So, even with that changed understanding of the first sentence, the conclusion is not invalidated (and the answer is not truly necessary).

I hope this helps!

Jeremy
 ally.ni
  • Posts: 11
  • Joined: Mar 09, 2021
|
#86382
Hello!

I want to clarify answer choice D. If we negate this answer choice it says that some conditions are treated effectively by medicine are also treated through the reduction of stress. However, this does not weaken the stimulus because this answer choice does not specify that the "some conditions" are conditions that can be treated only through the reduction of stress. Thus, answer choice D could be referring to any conditions and not necessarily the conditions mentioned in the stimulus.
 Rachael Wilkenfeld
PowerScore Staff
  • PowerScore Staff
  • Posts: 1358
  • Joined: Dec 15, 2011
|
#86406
Hi ally

The negation of D is a bit tricky. Let's think of the a more straightforward statement, then we can expand on the idea.

Some dogs are not fluffy.

The negation would be "No dogs are not fluffy."

Here, we have that some conditions treated effectively by meds are not also treated by stress reduction.

We'd negate that the same way---flip the some to a none/no.

No conditions treated effectively by meds are not also treated by stress reduction.

That doesn't weaken our conclusion that some high blood pressure cases must not be caused by stress. We don't know what the "some" cases in the conclusion are versus the "some in the premises." It doesn't really impact the stimulus.

Hope that helps!

Get the most out of your LSAT Prep Plus subscription.

Analyze and track your performance with our Testing and Analytics Package.